Maximize product of a vector with two vectors












2












$begingroup$


Say we're in a (complex) Hilbert space $H$ where we're given some two elements $a,bin H$ of norm 1.



The question is to find an element $psiin H$ (of norm 1) that maximizes $|langle psi,arangle langle b,psirangle|$ (or $|langle psi,arangle langle b,psirangle|^2$ if you prefer).



Mainly what I want to prove is one of the following inequalities:
$$
|langle psi,arangle langle b,psirangle|leq |langle a,b rangle|
$$

$$
|langle psi,arangle langle b,psirangle|leqbigg|frac{1+|langle a,brangle| }{2}bigg|
$$



Intuitively, my choice is to say that this maximizes when $psi = frac{a+b}{2}$, modulo the norm. And in fact one can get close to proving the inequalities by plugging that in. But I can't prove why this should be the case.



It is obvious that $psi$ should be in the span of $a,b$, since any component that's outside of this subspace will be wasted. So we can write $psi = xa + yb$ for some complex $x$ and $y$. But $a$ and $b$ need not be orthogonal, so it's not so simple.



Is there an inequality that one can use here to prove one of the above statements, or a quick way to find $psi$?










share|cite|improve this question











$endgroup$








  • 2




    $begingroup$
    Why don't you use $psi=x(a+b)+y(a-b)$ instead? The reason is that $(a+b)$ and $(a-b)$ are orthogonal.
    $endgroup$
    – Andrei
    Dec 20 '18 at 16:08






  • 1




    $begingroup$
    We have $langle a+b,a-brangle = 2mathop{Im}{langle b,arangle } $, and this doesn't have to be zero. In fact $frac{a+b}{2}$ isn't always the correct element that maximizes that product, there can be a rotation added to $a$ or $b$. At least that's what I think.
    $endgroup$
    – Kolja
    Dec 20 '18 at 17:26








  • 1




    $begingroup$
    All I see after a quick glance is that the first inequality is wrong. Take $H=mathbb C^2$, $a=(1,0)$, $b=(0,1)$ and $psi=frac1{sqrt{2}}(1,1)$. Then $frac12=|langle psi,aranglelangle b,psirangle|notleq |langle a,brangle|=0$. This however does not violate the second inequality you stated so that one has a good chance of being true maybe?
    $endgroup$
    – Frederik vom Ende
    Dec 21 '18 at 8:49








  • 1




    $begingroup$
    The first inequality counterexample was so obvious, I find it strange that I didn't think of it. Thanks !
    $endgroup$
    – Kolja
    Dec 23 '18 at 11:49
















2












$begingroup$


Say we're in a (complex) Hilbert space $H$ where we're given some two elements $a,bin H$ of norm 1.



The question is to find an element $psiin H$ (of norm 1) that maximizes $|langle psi,arangle langle b,psirangle|$ (or $|langle psi,arangle langle b,psirangle|^2$ if you prefer).



Mainly what I want to prove is one of the following inequalities:
$$
|langle psi,arangle langle b,psirangle|leq |langle a,b rangle|
$$

$$
|langle psi,arangle langle b,psirangle|leqbigg|frac{1+|langle a,brangle| }{2}bigg|
$$



Intuitively, my choice is to say that this maximizes when $psi = frac{a+b}{2}$, modulo the norm. And in fact one can get close to proving the inequalities by plugging that in. But I can't prove why this should be the case.



It is obvious that $psi$ should be in the span of $a,b$, since any component that's outside of this subspace will be wasted. So we can write $psi = xa + yb$ for some complex $x$ and $y$. But $a$ and $b$ need not be orthogonal, so it's not so simple.



Is there an inequality that one can use here to prove one of the above statements, or a quick way to find $psi$?










share|cite|improve this question











$endgroup$








  • 2




    $begingroup$
    Why don't you use $psi=x(a+b)+y(a-b)$ instead? The reason is that $(a+b)$ and $(a-b)$ are orthogonal.
    $endgroup$
    – Andrei
    Dec 20 '18 at 16:08






  • 1




    $begingroup$
    We have $langle a+b,a-brangle = 2mathop{Im}{langle b,arangle } $, and this doesn't have to be zero. In fact $frac{a+b}{2}$ isn't always the correct element that maximizes that product, there can be a rotation added to $a$ or $b$. At least that's what I think.
    $endgroup$
    – Kolja
    Dec 20 '18 at 17:26








  • 1




    $begingroup$
    All I see after a quick glance is that the first inequality is wrong. Take $H=mathbb C^2$, $a=(1,0)$, $b=(0,1)$ and $psi=frac1{sqrt{2}}(1,1)$. Then $frac12=|langle psi,aranglelangle b,psirangle|notleq |langle a,brangle|=0$. This however does not violate the second inequality you stated so that one has a good chance of being true maybe?
    $endgroup$
    – Frederik vom Ende
    Dec 21 '18 at 8:49








  • 1




    $begingroup$
    The first inequality counterexample was so obvious, I find it strange that I didn't think of it. Thanks !
    $endgroup$
    – Kolja
    Dec 23 '18 at 11:49














2












2








2





$begingroup$


Say we're in a (complex) Hilbert space $H$ where we're given some two elements $a,bin H$ of norm 1.



The question is to find an element $psiin H$ (of norm 1) that maximizes $|langle psi,arangle langle b,psirangle|$ (or $|langle psi,arangle langle b,psirangle|^2$ if you prefer).



Mainly what I want to prove is one of the following inequalities:
$$
|langle psi,arangle langle b,psirangle|leq |langle a,b rangle|
$$

$$
|langle psi,arangle langle b,psirangle|leqbigg|frac{1+|langle a,brangle| }{2}bigg|
$$



Intuitively, my choice is to say that this maximizes when $psi = frac{a+b}{2}$, modulo the norm. And in fact one can get close to proving the inequalities by plugging that in. But I can't prove why this should be the case.



It is obvious that $psi$ should be in the span of $a,b$, since any component that's outside of this subspace will be wasted. So we can write $psi = xa + yb$ for some complex $x$ and $y$. But $a$ and $b$ need not be orthogonal, so it's not so simple.



Is there an inequality that one can use here to prove one of the above statements, or a quick way to find $psi$?










share|cite|improve this question











$endgroup$




Say we're in a (complex) Hilbert space $H$ where we're given some two elements $a,bin H$ of norm 1.



The question is to find an element $psiin H$ (of norm 1) that maximizes $|langle psi,arangle langle b,psirangle|$ (or $|langle psi,arangle langle b,psirangle|^2$ if you prefer).



Mainly what I want to prove is one of the following inequalities:
$$
|langle psi,arangle langle b,psirangle|leq |langle a,b rangle|
$$

$$
|langle psi,arangle langle b,psirangle|leqbigg|frac{1+|langle a,brangle| }{2}bigg|
$$



Intuitively, my choice is to say that this maximizes when $psi = frac{a+b}{2}$, modulo the norm. And in fact one can get close to proving the inequalities by plugging that in. But I can't prove why this should be the case.



It is obvious that $psi$ should be in the span of $a,b$, since any component that's outside of this subspace will be wasted. So we can write $psi = xa + yb$ for some complex $x$ and $y$. But $a$ and $b$ need not be orthogonal, so it's not so simple.



Is there an inequality that one can use here to prove one of the above statements, or a quick way to find $psi$?







inequality hilbert-spaces inner-product-space cauchy-schwarz-inequality






share|cite|improve this question















share|cite|improve this question













share|cite|improve this question




share|cite|improve this question








edited Dec 20 '18 at 16:02







Kolja

















asked Dec 20 '18 at 15:54









KoljaKolja

590310




590310








  • 2




    $begingroup$
    Why don't you use $psi=x(a+b)+y(a-b)$ instead? The reason is that $(a+b)$ and $(a-b)$ are orthogonal.
    $endgroup$
    – Andrei
    Dec 20 '18 at 16:08






  • 1




    $begingroup$
    We have $langle a+b,a-brangle = 2mathop{Im}{langle b,arangle } $, and this doesn't have to be zero. In fact $frac{a+b}{2}$ isn't always the correct element that maximizes that product, there can be a rotation added to $a$ or $b$. At least that's what I think.
    $endgroup$
    – Kolja
    Dec 20 '18 at 17:26








  • 1




    $begingroup$
    All I see after a quick glance is that the first inequality is wrong. Take $H=mathbb C^2$, $a=(1,0)$, $b=(0,1)$ and $psi=frac1{sqrt{2}}(1,1)$. Then $frac12=|langle psi,aranglelangle b,psirangle|notleq |langle a,brangle|=0$. This however does not violate the second inequality you stated so that one has a good chance of being true maybe?
    $endgroup$
    – Frederik vom Ende
    Dec 21 '18 at 8:49








  • 1




    $begingroup$
    The first inequality counterexample was so obvious, I find it strange that I didn't think of it. Thanks !
    $endgroup$
    – Kolja
    Dec 23 '18 at 11:49














  • 2




    $begingroup$
    Why don't you use $psi=x(a+b)+y(a-b)$ instead? The reason is that $(a+b)$ and $(a-b)$ are orthogonal.
    $endgroup$
    – Andrei
    Dec 20 '18 at 16:08






  • 1




    $begingroup$
    We have $langle a+b,a-brangle = 2mathop{Im}{langle b,arangle } $, and this doesn't have to be zero. In fact $frac{a+b}{2}$ isn't always the correct element that maximizes that product, there can be a rotation added to $a$ or $b$. At least that's what I think.
    $endgroup$
    – Kolja
    Dec 20 '18 at 17:26








  • 1




    $begingroup$
    All I see after a quick glance is that the first inequality is wrong. Take $H=mathbb C^2$, $a=(1,0)$, $b=(0,1)$ and $psi=frac1{sqrt{2}}(1,1)$. Then $frac12=|langle psi,aranglelangle b,psirangle|notleq |langle a,brangle|=0$. This however does not violate the second inequality you stated so that one has a good chance of being true maybe?
    $endgroup$
    – Frederik vom Ende
    Dec 21 '18 at 8:49








  • 1




    $begingroup$
    The first inequality counterexample was so obvious, I find it strange that I didn't think of it. Thanks !
    $endgroup$
    – Kolja
    Dec 23 '18 at 11:49








2




2




$begingroup$
Why don't you use $psi=x(a+b)+y(a-b)$ instead? The reason is that $(a+b)$ and $(a-b)$ are orthogonal.
$endgroup$
– Andrei
Dec 20 '18 at 16:08




$begingroup$
Why don't you use $psi=x(a+b)+y(a-b)$ instead? The reason is that $(a+b)$ and $(a-b)$ are orthogonal.
$endgroup$
– Andrei
Dec 20 '18 at 16:08




1




1




$begingroup$
We have $langle a+b,a-brangle = 2mathop{Im}{langle b,arangle } $, and this doesn't have to be zero. In fact $frac{a+b}{2}$ isn't always the correct element that maximizes that product, there can be a rotation added to $a$ or $b$. At least that's what I think.
$endgroup$
– Kolja
Dec 20 '18 at 17:26






$begingroup$
We have $langle a+b,a-brangle = 2mathop{Im}{langle b,arangle } $, and this doesn't have to be zero. In fact $frac{a+b}{2}$ isn't always the correct element that maximizes that product, there can be a rotation added to $a$ or $b$. At least that's what I think.
$endgroup$
– Kolja
Dec 20 '18 at 17:26






1




1




$begingroup$
All I see after a quick glance is that the first inequality is wrong. Take $H=mathbb C^2$, $a=(1,0)$, $b=(0,1)$ and $psi=frac1{sqrt{2}}(1,1)$. Then $frac12=|langle psi,aranglelangle b,psirangle|notleq |langle a,brangle|=0$. This however does not violate the second inequality you stated so that one has a good chance of being true maybe?
$endgroup$
– Frederik vom Ende
Dec 21 '18 at 8:49






$begingroup$
All I see after a quick glance is that the first inequality is wrong. Take $H=mathbb C^2$, $a=(1,0)$, $b=(0,1)$ and $psi=frac1{sqrt{2}}(1,1)$. Then $frac12=|langle psi,aranglelangle b,psirangle|notleq |langle a,brangle|=0$. This however does not violate the second inequality you stated so that one has a good chance of being true maybe?
$endgroup$
– Frederik vom Ende
Dec 21 '18 at 8:49






1




1




$begingroup$
The first inequality counterexample was so obvious, I find it strange that I didn't think of it. Thanks !
$endgroup$
– Kolja
Dec 23 '18 at 11:49




$begingroup$
The first inequality counterexample was so obvious, I find it strange that I didn't think of it. Thanks !
$endgroup$
– Kolja
Dec 23 '18 at 11:49










1 Answer
1






active

oldest

votes


















1












$begingroup$

What you consider here is the numerical radius of the rank-1 operator $A:Hto H$, $hmapsto langle b,hrangle a$. To put things into context, the numerical range of $A$ is given by
$$
W(A)overset{text{Def.}}=lbracelangle psi,Apsirangle,|,psiin H,|psi|_H=1rbrace=lbracelangle psi,aranglelangle b,psirangle,|,psiin H,|psi|_H=1rbrace
$$

and the numerical radius of $A$ by
$$
r(A)=suplbrace |lambda|,|,lambdain W(A)rbrace=sup_{psiin H,|psi|_H=1}|langle psi,aranglelangle b,psirangle|,.
$$

As $A$ is of rank one, Lemma 2.1 in this paper (click here for accessible pdf) yields
$$
r(A)=frac{|A|+|operatorname{tr}(A)|}2,.
$$

One readily verifies that $A$ has operator norm $1$ and $operatorname{tr}(A)=langle b,arangle$ so
$$
|langlepsi,aranglelangle b,psirangle|leq sup_{psiin H,|psi|_H=1}|langle psi,aranglelangle b,psirangle|=r(A)=frac{1+|langle a,brangle|}2
$$

for all $psiin H,|psi|_H=1$ which is the desired inequality.






share|cite|improve this answer









$endgroup$













    Your Answer





    StackExchange.ifUsing("editor", function () {
    return StackExchange.using("mathjaxEditing", function () {
    StackExchange.MarkdownEditor.creationCallbacks.add(function (editor, postfix) {
    StackExchange.mathjaxEditing.prepareWmdForMathJax(editor, postfix, [["$", "$"], ["\\(","\\)"]]);
    });
    });
    }, "mathjax-editing");

    StackExchange.ready(function() {
    var channelOptions = {
    tags: "".split(" "),
    id: "69"
    };
    initTagRenderer("".split(" "), "".split(" "), channelOptions);

    StackExchange.using("externalEditor", function() {
    // Have to fire editor after snippets, if snippets enabled
    if (StackExchange.settings.snippets.snippetsEnabled) {
    StackExchange.using("snippets", function() {
    createEditor();
    });
    }
    else {
    createEditor();
    }
    });

    function createEditor() {
    StackExchange.prepareEditor({
    heartbeatType: 'answer',
    autoActivateHeartbeat: false,
    convertImagesToLinks: true,
    noModals: true,
    showLowRepImageUploadWarning: true,
    reputationToPostImages: 10,
    bindNavPrevention: true,
    postfix: "",
    imageUploader: {
    brandingHtml: "Powered by u003ca class="icon-imgur-white" href="https://imgur.com/"u003eu003c/au003e",
    contentPolicyHtml: "User contributions licensed under u003ca href="https://creativecommons.org/licenses/by-sa/3.0/"u003ecc by-sa 3.0 with attribution requiredu003c/au003e u003ca href="https://stackoverflow.com/legal/content-policy"u003e(content policy)u003c/au003e",
    allowUrls: true
    },
    noCode: true, onDemand: true,
    discardSelector: ".discard-answer"
    ,immediatelyShowMarkdownHelp:true
    });


    }
    });














    draft saved

    draft discarded


















    StackExchange.ready(
    function () {
    StackExchange.openid.initPostLogin('.new-post-login', 'https%3a%2f%2fmath.stackexchange.com%2fquestions%2f3047678%2fmaximize-product-of-a-vector-with-two-vectors%23new-answer', 'question_page');
    }
    );

    Post as a guest















    Required, but never shown

























    1 Answer
    1






    active

    oldest

    votes








    1 Answer
    1






    active

    oldest

    votes









    active

    oldest

    votes






    active

    oldest

    votes









    1












    $begingroup$

    What you consider here is the numerical radius of the rank-1 operator $A:Hto H$, $hmapsto langle b,hrangle a$. To put things into context, the numerical range of $A$ is given by
    $$
    W(A)overset{text{Def.}}=lbracelangle psi,Apsirangle,|,psiin H,|psi|_H=1rbrace=lbracelangle psi,aranglelangle b,psirangle,|,psiin H,|psi|_H=1rbrace
    $$

    and the numerical radius of $A$ by
    $$
    r(A)=suplbrace |lambda|,|,lambdain W(A)rbrace=sup_{psiin H,|psi|_H=1}|langle psi,aranglelangle b,psirangle|,.
    $$

    As $A$ is of rank one, Lemma 2.1 in this paper (click here for accessible pdf) yields
    $$
    r(A)=frac{|A|+|operatorname{tr}(A)|}2,.
    $$

    One readily verifies that $A$ has operator norm $1$ and $operatorname{tr}(A)=langle b,arangle$ so
    $$
    |langlepsi,aranglelangle b,psirangle|leq sup_{psiin H,|psi|_H=1}|langle psi,aranglelangle b,psirangle|=r(A)=frac{1+|langle a,brangle|}2
    $$

    for all $psiin H,|psi|_H=1$ which is the desired inequality.






    share|cite|improve this answer









    $endgroup$


















      1












      $begingroup$

      What you consider here is the numerical radius of the rank-1 operator $A:Hto H$, $hmapsto langle b,hrangle a$. To put things into context, the numerical range of $A$ is given by
      $$
      W(A)overset{text{Def.}}=lbracelangle psi,Apsirangle,|,psiin H,|psi|_H=1rbrace=lbracelangle psi,aranglelangle b,psirangle,|,psiin H,|psi|_H=1rbrace
      $$

      and the numerical radius of $A$ by
      $$
      r(A)=suplbrace |lambda|,|,lambdain W(A)rbrace=sup_{psiin H,|psi|_H=1}|langle psi,aranglelangle b,psirangle|,.
      $$

      As $A$ is of rank one, Lemma 2.1 in this paper (click here for accessible pdf) yields
      $$
      r(A)=frac{|A|+|operatorname{tr}(A)|}2,.
      $$

      One readily verifies that $A$ has operator norm $1$ and $operatorname{tr}(A)=langle b,arangle$ so
      $$
      |langlepsi,aranglelangle b,psirangle|leq sup_{psiin H,|psi|_H=1}|langle psi,aranglelangle b,psirangle|=r(A)=frac{1+|langle a,brangle|}2
      $$

      for all $psiin H,|psi|_H=1$ which is the desired inequality.






      share|cite|improve this answer









      $endgroup$
















        1












        1








        1





        $begingroup$

        What you consider here is the numerical radius of the rank-1 operator $A:Hto H$, $hmapsto langle b,hrangle a$. To put things into context, the numerical range of $A$ is given by
        $$
        W(A)overset{text{Def.}}=lbracelangle psi,Apsirangle,|,psiin H,|psi|_H=1rbrace=lbracelangle psi,aranglelangle b,psirangle,|,psiin H,|psi|_H=1rbrace
        $$

        and the numerical radius of $A$ by
        $$
        r(A)=suplbrace |lambda|,|,lambdain W(A)rbrace=sup_{psiin H,|psi|_H=1}|langle psi,aranglelangle b,psirangle|,.
        $$

        As $A$ is of rank one, Lemma 2.1 in this paper (click here for accessible pdf) yields
        $$
        r(A)=frac{|A|+|operatorname{tr}(A)|}2,.
        $$

        One readily verifies that $A$ has operator norm $1$ and $operatorname{tr}(A)=langle b,arangle$ so
        $$
        |langlepsi,aranglelangle b,psirangle|leq sup_{psiin H,|psi|_H=1}|langle psi,aranglelangle b,psirangle|=r(A)=frac{1+|langle a,brangle|}2
        $$

        for all $psiin H,|psi|_H=1$ which is the desired inequality.






        share|cite|improve this answer









        $endgroup$



        What you consider here is the numerical radius of the rank-1 operator $A:Hto H$, $hmapsto langle b,hrangle a$. To put things into context, the numerical range of $A$ is given by
        $$
        W(A)overset{text{Def.}}=lbracelangle psi,Apsirangle,|,psiin H,|psi|_H=1rbrace=lbracelangle psi,aranglelangle b,psirangle,|,psiin H,|psi|_H=1rbrace
        $$

        and the numerical radius of $A$ by
        $$
        r(A)=suplbrace |lambda|,|,lambdain W(A)rbrace=sup_{psiin H,|psi|_H=1}|langle psi,aranglelangle b,psirangle|,.
        $$

        As $A$ is of rank one, Lemma 2.1 in this paper (click here for accessible pdf) yields
        $$
        r(A)=frac{|A|+|operatorname{tr}(A)|}2,.
        $$

        One readily verifies that $A$ has operator norm $1$ and $operatorname{tr}(A)=langle b,arangle$ so
        $$
        |langlepsi,aranglelangle b,psirangle|leq sup_{psiin H,|psi|_H=1}|langle psi,aranglelangle b,psirangle|=r(A)=frac{1+|langle a,brangle|}2
        $$

        for all $psiin H,|psi|_H=1$ which is the desired inequality.







        share|cite|improve this answer












        share|cite|improve this answer



        share|cite|improve this answer










        answered Dec 21 '18 at 11:50









        Frederik vom EndeFrederik vom Ende

        9821322




        9821322






























            draft saved

            draft discarded




















































            Thanks for contributing an answer to Mathematics Stack Exchange!


            • Please be sure to answer the question. Provide details and share your research!

            But avoid



            • Asking for help, clarification, or responding to other answers.

            • Making statements based on opinion; back them up with references or personal experience.


            Use MathJax to format equations. MathJax reference.


            To learn more, see our tips on writing great answers.




            draft saved


            draft discarded














            StackExchange.ready(
            function () {
            StackExchange.openid.initPostLogin('.new-post-login', 'https%3a%2f%2fmath.stackexchange.com%2fquestions%2f3047678%2fmaximize-product-of-a-vector-with-two-vectors%23new-answer', 'question_page');
            }
            );

            Post as a guest















            Required, but never shown





















































            Required, but never shown














            Required, but never shown












            Required, but never shown







            Required, but never shown

































            Required, but never shown














            Required, but never shown












            Required, but never shown







            Required, but never shown







            Popular posts from this blog

            Aardman Animations

            Are they similar matrix

            Ficheiro:Flag of Oman.svg